Sum of fraction series












0












$begingroup$


I have been trying to solve this question but I am not sure how to, and would appreciate some help. Thanks.




Simplify:
$$frac{1}{4} +frac{1}{4^2} +frac{4^2}{4^3} +frac{4^2}{4^4} +frac{4^4}{4^5} +frac{4^4}{4^6} +ldots +frac{4^{102}}{4^{103}} +frac{4^{102}}{4^{104}}.$$











share|cite|improve this question











$endgroup$












  • $begingroup$
    Jimmy: what are the first two terms, and the next two and so on? Can you sum a geometric series?
    $endgroup$
    – J.G.
    Dec 8 '18 at 14:21






  • 4




    $begingroup$
    You're just repeatedly adding $frac{1}{4}$ and $frac{1}{16}$...
    $endgroup$
    – KM101
    Dec 8 '18 at 14:23


















0












$begingroup$


I have been trying to solve this question but I am not sure how to, and would appreciate some help. Thanks.




Simplify:
$$frac{1}{4} +frac{1}{4^2} +frac{4^2}{4^3} +frac{4^2}{4^4} +frac{4^4}{4^5} +frac{4^4}{4^6} +ldots +frac{4^{102}}{4^{103}} +frac{4^{102}}{4^{104}}.$$











share|cite|improve this question











$endgroup$












  • $begingroup$
    Jimmy: what are the first two terms, and the next two and so on? Can you sum a geometric series?
    $endgroup$
    – J.G.
    Dec 8 '18 at 14:21






  • 4




    $begingroup$
    You're just repeatedly adding $frac{1}{4}$ and $frac{1}{16}$...
    $endgroup$
    – KM101
    Dec 8 '18 at 14:23
















0












0








0





$begingroup$


I have been trying to solve this question but I am not sure how to, and would appreciate some help. Thanks.




Simplify:
$$frac{1}{4} +frac{1}{4^2} +frac{4^2}{4^3} +frac{4^2}{4^4} +frac{4^4}{4^5} +frac{4^4}{4^6} +ldots +frac{4^{102}}{4^{103}} +frac{4^{102}}{4^{104}}.$$











share|cite|improve this question











$endgroup$




I have been trying to solve this question but I am not sure how to, and would appreciate some help. Thanks.




Simplify:
$$frac{1}{4} +frac{1}{4^2} +frac{4^2}{4^3} +frac{4^2}{4^4} +frac{4^4}{4^5} +frac{4^4}{4^6} +ldots +frac{4^{102}}{4^{103}} +frac{4^{102}}{4^{104}}.$$








fractions






share|cite|improve this question















share|cite|improve this question













share|cite|improve this question




share|cite|improve this question








edited Dec 8 '18 at 14:19









Rócherz

2,7762721




2,7762721










asked Dec 8 '18 at 14:16









UnknownUnknown

73




73












  • $begingroup$
    Jimmy: what are the first two terms, and the next two and so on? Can you sum a geometric series?
    $endgroup$
    – J.G.
    Dec 8 '18 at 14:21






  • 4




    $begingroup$
    You're just repeatedly adding $frac{1}{4}$ and $frac{1}{16}$...
    $endgroup$
    – KM101
    Dec 8 '18 at 14:23




















  • $begingroup$
    Jimmy: what are the first two terms, and the next two and so on? Can you sum a geometric series?
    $endgroup$
    – J.G.
    Dec 8 '18 at 14:21






  • 4




    $begingroup$
    You're just repeatedly adding $frac{1}{4}$ and $frac{1}{16}$...
    $endgroup$
    – KM101
    Dec 8 '18 at 14:23


















$begingroup$
Jimmy: what are the first two terms, and the next two and so on? Can you sum a geometric series?
$endgroup$
– J.G.
Dec 8 '18 at 14:21




$begingroup$
Jimmy: what are the first two terms, and the next two and so on? Can you sum a geometric series?
$endgroup$
– J.G.
Dec 8 '18 at 14:21




4




4




$begingroup$
You're just repeatedly adding $frac{1}{4}$ and $frac{1}{16}$...
$endgroup$
– KM101
Dec 8 '18 at 14:23






$begingroup$
You're just repeatedly adding $frac{1}{4}$ and $frac{1}{16}$...
$endgroup$
– KM101
Dec 8 '18 at 14:23












1 Answer
1






active

oldest

votes


















0












$begingroup$

The required sum is $$sum_{r=1}^{52}Bigg(Big(frac{4^{2r-2}}{4^{2r-1}}Big)+Big(frac{4^{2r-2}}{4^{2r}}Big)Bigg)$$
$$=sum_{r=1}^{52}Bigg(Big(frac{1}{4}Big)+Big(frac{1}{4^2}Big)Bigg)$$
$$=52Bigg(frac{1}{4}+frac{1}{4^2}Bigg)$$
$$=13+frac{13}{4}$$
$$=frac{65}{4}$$
Hope it is helpful:)






share|cite|improve this answer









$endgroup$













    Your Answer





    StackExchange.ifUsing("editor", function () {
    return StackExchange.using("mathjaxEditing", function () {
    StackExchange.MarkdownEditor.creationCallbacks.add(function (editor, postfix) {
    StackExchange.mathjaxEditing.prepareWmdForMathJax(editor, postfix, [["$", "$"], ["\\(","\\)"]]);
    });
    });
    }, "mathjax-editing");

    StackExchange.ready(function() {
    var channelOptions = {
    tags: "".split(" "),
    id: "69"
    };
    initTagRenderer("".split(" "), "".split(" "), channelOptions);

    StackExchange.using("externalEditor", function() {
    // Have to fire editor after snippets, if snippets enabled
    if (StackExchange.settings.snippets.snippetsEnabled) {
    StackExchange.using("snippets", function() {
    createEditor();
    });
    }
    else {
    createEditor();
    }
    });

    function createEditor() {
    StackExchange.prepareEditor({
    heartbeatType: 'answer',
    autoActivateHeartbeat: false,
    convertImagesToLinks: true,
    noModals: true,
    showLowRepImageUploadWarning: true,
    reputationToPostImages: 10,
    bindNavPrevention: true,
    postfix: "",
    imageUploader: {
    brandingHtml: "Powered by u003ca class="icon-imgur-white" href="https://imgur.com/"u003eu003c/au003e",
    contentPolicyHtml: "User contributions licensed under u003ca href="https://creativecommons.org/licenses/by-sa/3.0/"u003ecc by-sa 3.0 with attribution requiredu003c/au003e u003ca href="https://stackoverflow.com/legal/content-policy"u003e(content policy)u003c/au003e",
    allowUrls: true
    },
    noCode: true, onDemand: true,
    discardSelector: ".discard-answer"
    ,immediatelyShowMarkdownHelp:true
    });


    }
    });














    draft saved

    draft discarded


















    StackExchange.ready(
    function () {
    StackExchange.openid.initPostLogin('.new-post-login', 'https%3a%2f%2fmath.stackexchange.com%2fquestions%2f3031151%2fsum-of-fraction-series%23new-answer', 'question_page');
    }
    );

    Post as a guest















    Required, but never shown

























    1 Answer
    1






    active

    oldest

    votes








    1 Answer
    1






    active

    oldest

    votes









    active

    oldest

    votes






    active

    oldest

    votes









    0












    $begingroup$

    The required sum is $$sum_{r=1}^{52}Bigg(Big(frac{4^{2r-2}}{4^{2r-1}}Big)+Big(frac{4^{2r-2}}{4^{2r}}Big)Bigg)$$
    $$=sum_{r=1}^{52}Bigg(Big(frac{1}{4}Big)+Big(frac{1}{4^2}Big)Bigg)$$
    $$=52Bigg(frac{1}{4}+frac{1}{4^2}Bigg)$$
    $$=13+frac{13}{4}$$
    $$=frac{65}{4}$$
    Hope it is helpful:)






    share|cite|improve this answer









    $endgroup$


















      0












      $begingroup$

      The required sum is $$sum_{r=1}^{52}Bigg(Big(frac{4^{2r-2}}{4^{2r-1}}Big)+Big(frac{4^{2r-2}}{4^{2r}}Big)Bigg)$$
      $$=sum_{r=1}^{52}Bigg(Big(frac{1}{4}Big)+Big(frac{1}{4^2}Big)Bigg)$$
      $$=52Bigg(frac{1}{4}+frac{1}{4^2}Bigg)$$
      $$=13+frac{13}{4}$$
      $$=frac{65}{4}$$
      Hope it is helpful:)






      share|cite|improve this answer









      $endgroup$
















        0












        0








        0





        $begingroup$

        The required sum is $$sum_{r=1}^{52}Bigg(Big(frac{4^{2r-2}}{4^{2r-1}}Big)+Big(frac{4^{2r-2}}{4^{2r}}Big)Bigg)$$
        $$=sum_{r=1}^{52}Bigg(Big(frac{1}{4}Big)+Big(frac{1}{4^2}Big)Bigg)$$
        $$=52Bigg(frac{1}{4}+frac{1}{4^2}Bigg)$$
        $$=13+frac{13}{4}$$
        $$=frac{65}{4}$$
        Hope it is helpful:)






        share|cite|improve this answer









        $endgroup$



        The required sum is $$sum_{r=1}^{52}Bigg(Big(frac{4^{2r-2}}{4^{2r-1}}Big)+Big(frac{4^{2r-2}}{4^{2r}}Big)Bigg)$$
        $$=sum_{r=1}^{52}Bigg(Big(frac{1}{4}Big)+Big(frac{1}{4^2}Big)Bigg)$$
        $$=52Bigg(frac{1}{4}+frac{1}{4^2}Bigg)$$
        $$=13+frac{13}{4}$$
        $$=frac{65}{4}$$
        Hope it is helpful:)







        share|cite|improve this answer












        share|cite|improve this answer



        share|cite|improve this answer










        answered Dec 8 '18 at 14:53









        MartundMartund

        1,593212




        1,593212






























            draft saved

            draft discarded




















































            Thanks for contributing an answer to Mathematics Stack Exchange!


            • Please be sure to answer the question. Provide details and share your research!

            But avoid



            • Asking for help, clarification, or responding to other answers.

            • Making statements based on opinion; back them up with references or personal experience.


            Use MathJax to format equations. MathJax reference.


            To learn more, see our tips on writing great answers.




            draft saved


            draft discarded














            StackExchange.ready(
            function () {
            StackExchange.openid.initPostLogin('.new-post-login', 'https%3a%2f%2fmath.stackexchange.com%2fquestions%2f3031151%2fsum-of-fraction-series%23new-answer', 'question_page');
            }
            );

            Post as a guest















            Required, but never shown





















































            Required, but never shown














            Required, but never shown












            Required, but never shown







            Required, but never shown

































            Required, but never shown














            Required, but never shown












            Required, but never shown







            Required, but never shown







            Popular posts from this blog

            Quarter-circle Tiles

            build a pushdown automaton that recognizes the reverse language of a given pushdown automaton?

            Mont Emei